TransWikia.com

Transition probability between two harmonic oscillator states (Hemite polynomial integration)

Physics Asked on January 8, 2021

Objective:

Show that

$$
int^{infty}_{-infty} x e^{-x^2} H_n(x) H_m(x) dx = pi^{1/2} 2^{n-1} n! delta_{m,n-1} + pi^{1/2} 2^n (n+1)! delta_{m,n+1}
$$

My attempt at this is:


begin{eqnarray*}
sum^infty_{n=0} frac{2^n s^n u^n}{n!} (s+u) sqrt{pi} &=& sum^infty_{m,n=0}frac{s^m u^n}{m! n!} int x H_m(x) H_n(x) e^{-x^2} dx
sqrt{pi}bigg( sum^infty_{n=0} frac{2^n s^{n+1} u^n}{n!} +sum^infty_{n=0} frac{2^n s^n u^{n+1}}{n!} bigg) &=& sum^infty_{m,n=0}frac{s^m u^n}{m! n!} int x H_m(x) H_n(x) e^{-x^2} dx
end{eqnarray*}

I’m reasonably confident up to here.

begin{eqnarray*}
sqrt{pi}bigg( sum^infty_{n=0} frac{2^n s^{n+1} u^n}{n!} +sum^infty_{n=0} frac{2^n s^n u^{n+1}}{n!} bigg) delta_{m,n+1}&=& delta_{m,n+1}sum^infty_{m,n=0}frac{s^m u^n}{m! n!} int x H_m(x) H_n(x) e^{-x^2} dx
end{eqnarray*}

begin{eqnarray*}
sqrt{pi}bigg( sum^infty_{n=0} frac{2^n s^{n+1} u^n}{n!}big(delta_{m,n+1}frac{m! n!}{s^m u^n}big) +sum^infty_{n=0} frac{2^n s^n u^{n+1}}{n!}big(delta_{m,n+1}frac{m! n!}{s^m u^n}big) bigg) delta_{m,n+1}&=& int x H_m(x) H_n(x) e^{-x^2} dx
end{eqnarray*}

Then I thought maybe I could try re-indexing the n and m in the second $big(delta_{m,n+1}frac{m! n!}{s^m u^n}big)$ term.

begin{eqnarray*}
sqrt{pi}bigg( sum^infty_{n=0} frac{2^n s^{n+1} u^n}{n!}big(frac{(n+1)! n!}{s^{n+1} u^n}big) +sum^infty_{n=0} frac{2^n s^n u^{n+1}}{n!}big(delta_{m,n+1}frac{n! m!}{s^n u^m}big) bigg) delta_{m,n+1}&=& int x H_m(x) H_n(x) e^{-x^2} dx
bigg(sqrt{pi} 2^n (n+1)! + sqrt{pi} sum^infty_{n=0} frac{2^n s^n u^{n+1}}{n!}big(delta_{m,n+1}frac{n! m!}{s^n u^m}big) bigg) delta_{m,n+1}&=& int x H_m(x) H_n(x) e^{-x^2} dx
sqrt{pi} 2^n (n+1)! delta_{m,n+1} + sqrt{pi} sum^infty_{n=0} frac{2^n s^n u^{n+1}}{n!}big(frac{n! (n+1)!}{s^n u^{n+1}}big) delta_{m,n+1}&=& int x H_m(x) H_n(x) e^{-x^2} dx
sqrt{pi} 2^n (n+1)! delta_{m,n+1} + sqrt{pi} 2^n (n+1)! delta_{m,n+1}&=& int x H_m(x) H_n(x) e^{-x^2} dx
sqrt{pi} 2^{n+1} (n+1)! delta_{m,n+1} &=& int x H_m(x) H_n(x) e^{-x^2} dx
end{eqnarray*}


Obviously the wrong answer. I got the following hint from a friend: "You should take d^n/du^n d^m/ds^m (n-th and m-th derivatives) of the second line, answer has two different Kronecker deltas."

Can anyone specifically help me figure out how to get this $delta_{m,n-1}$ factor? I can’t figure out that one.

One Answer

@secavara's hint is, in fact, a one liner, if only you used Hermite functions, which have a flat measure, $int_{-infty}^infty psi_n(x) psi_m(x) ,dx = delta_{nm}$, $$psi_n(x) = left (2^n n! sqrt{pi} right )^{-frac12} e^{-frac{x^2}{2}} H_n(x) = (-1)^n left (2^n n! sqrt{pi} right)^{-frac12} e^{frac{x^2}{2}} ~ partial_x^n~ e^{-x^2}, sqrt{2(n+1)}~~psi_{n+1}(x)= left ( x- partial_x right ) psi_n(x). $$

Consequently, $$ partial_x(psi_npsi_m)= 2xpsi_npsi_m -sqrt{2(n+1)}~ psi_{n+1}psi_m -sqrt{2(m+1)} ~psi_npsi_{m+1}. $$

Answered by Cosmas Zachos on January 8, 2021

Add your own answers!

Ask a Question

Get help from others!

© 2024 TransWikia.com. All rights reserved. Sites we Love: PCI Database, UKBizDB, Menu Kuliner, Sharing RPP